Difference between revisions of "2003 AIME I Problems/Problem 14"

(See also)
m
Line 3: Line 3:
  
 
== Solution ==
 
== Solution ==
 
+
{{solution}}
 
== See also ==
 
== See also ==
 
* [[2003 AIME I Problems/Problem 13 | Previous problem]]
 
* [[2003 AIME I Problems/Problem 13 | Previous problem]]
 
* [[2003 AIME I Problems/Problem 15 | Next problem]]
 
* [[2003 AIME I Problems/Problem 15 | Next problem]]
 
* [[2003 AIME I Problems]]
 
* [[2003 AIME I Problems]]
 +
 +
[[Category:Intermediate Number Theory Problems]]

Revision as of 23:23, 4 November 2006

Problem

The decimal representation of $m/n,$ where $m$ and $n$ are relatively prime positive integers and $m < n,$ contains the digits 2, 5, and 1 consecutively, and in that order. Find the smallest value of $n$ for which this is possible.

Solution

This problem needs a solution. If you have a solution for it, please help us out by adding it.

See also